LSAT and Law School Admissions Forum

Get expert LSAT preparation and law school admissions advice from PowerScore Test Preparation.

User avatar
 Dave Killoran
PowerScore Staff
  • PowerScore Staff
  • Posts: 5852
  • Joined: Mar 25, 2011
|
#41612
Complete Question Explanation
(The complete setup for this game can be found here: lsat/viewtopic.php?t=14535)

The correct answer is (A)

The question stem, in combination with one of the key deductions, establishes a double arrow where:


..... ..... ..... ..... ..... ..... ..... P ..... :dbl: ..... U


Of course, if U and P must always be together, then via the contrapositive, R and N must also always be together:

..... ..... ..... ..... ..... ..... ..... R ..... :dbl: ..... N


From this we can conclude that floor 1 includes R cars, and adding another of the key deductions, we can conclude that floor 1 includes F cars:
D91_Game_#3_#19_diagram 1.png
Consequently, answer choice (A) is correct.
You do not have the required permissions to view the files attached to this post.
 kingsnqueens
  • Posts: 3
  • Joined: Oct 15, 2017
|
#40859
Hello,

Just wondering whether E is only incorrect because it was already established in the initial diagram or is there another reason?
 Claire Horan
PowerScore Staff
  • PowerScore Staff
  • Posts: 408
  • Joined: Apr 18, 2016
|
#40893
Hi Kingsnqueens,

First, a correct statement in logic games will never be an an incorrect answer for the reason you suggested, that it was given to you in the rules.

This is a must be true question, so any statement that is determined by the rules and by any constraints given in the local questions would be a correct answer.

Now looking at answer choice (E), is it incorrect because there may not be sports cars on floor 3. We know that floor 1 must have new, research model family cars and that floor 3 must have used, production model cars. But the production model cars do not have to be sports cars. There may be family cars on that floor.

There is a rule that reads: "If the exhibition includes both family cars and sports cars, then each family car is displayed on a lower-numbered floor than any sports car." But there is no requirement that the exhibition include both family and sports cars. The exhibition may be three floors of family cars.

Please let us know if you have any more questions, and good luck!

Claire
 medialaw111516
  • Posts: 80
  • Joined: Dec 11, 2018
|
#71144
Hi,

Still having difficulty understanding why A is correct over E.
User avatar
 KelseyWoods
PowerScore Staff
  • PowerScore Staff
  • Posts: 1079
  • Joined: Jun 26, 2013
|
#71161
Hi MediaLaw!

It would help to know more about where your confusion lies but I'll do my best to clarify! Answer choice (A) is correct because if all of the production models are used, then the cars on Floor 1 cannot be production models, because they are new.

Essentially, the rule introduced in the question stem would be:

P :arrow: U

The contrapositive would be:

U :arrow: P

Since there are only 2 options for everything (U=N P=R), we could re-write the contrapositive as:

N :arrow: R

So since we know from our Global Rules that Floor 1 has N, then Floor 1 must also have R.

We also have a Global Rule that tells us that no research models are sports cars. One way to diagram that is:

R :arrow: S

In other words, if it's a research model, it cannot be a sports car. Since S=F we can rewrite that rule as:

R :arrow: F

Back to our local question, since we figured out that Floor 1 must have R, that means Floor 1 must also have F.

That gives us answer choice (A).

If you're tempted by answer choice (E), I'm guessing it's because you made a Mistaken Reversal inference in your Global Diagram. From our main diagram, we know that the cars on Floor 3 are U and P. But we do NOT know if the cars on Floor 3 are S or F. Research models cannot be sports cars, so if we have a sports car, it must be a production model (S :arrow: P). But that is NOT the same as saying that if we have a production model, it has to be a sports car. A sports car can ONLY be a production model, but a production model could be either a sports car or a family car. So on Floor 3, knowing that we have a production model does not tell us whether we have sports care or a family car.

Watch out for those Mistaken Reversals!

Hope this helps!

Best,
Kelsey
 lolaSur
  • Posts: 46
  • Joined: Nov 11, 2019
|
#72239
I read the explanation for this game, but I am still confused by question 19. Could you please go over the steps of how to solve question 19 which states "if all the production models in the exhibition are used then which one of the following statements must be true?"

I don't understand what is meant by "if all the production models in the exhibition are used" since we don't know exactly how many production models there are in total.

This is a local question, so we have to diagram right?


Thank you in advance!
 lolaSur
  • Posts: 46
  • Joined: Nov 11, 2019
|
#72241
Hi!

Could you also please go over question 19 of this game. The question states "if all the production models in the exhibition are used then which one of the following statements must be true?"

I don't understand what is meant by "if all the production models are used." The game does not provide a number of how many production models exist.

Also, the explanations state that the double arrow P :dbl: U and R :dbl: N is established. How is this possible?

I understand how when listing the rules we arrived at
U :dblline: R and R :dblline: S

U :dblline: R can also be U :arrow: P and if R :arrow: N is this correct? so how do we arrive at P :dbl: U and R :dbl: N for question 19?
User avatar
 Stephanie Oswalt
PowerScore Staff
  • PowerScore Staff
  • Posts: 811
  • Joined: Jan 11, 2016
|
#72255
Hi Lola,

I moved your posts to the thread discussing #19 (you posted in the "setup" section — we actually offer an overall setup and an explanation of each question :D ). Please review the above explanations, and let us know if that helps! Thanks! :)
 lolaSur
  • Posts: 46
  • Joined: Nov 11, 2019
|
#73666
Thank you! I misinterpreted the substitution rule. I thought it meant that I had to use P, to fill all possible spots for P. I see now this is a substitution rule and I should have interpreted it as p :arrow: u. I also understand that the contrapositive is n :arrow: r. Thank you again!
User avatar
 ridolph.lauren
  • Posts: 16
  • Joined: Feb 20, 2022
|
#96879
Hello,

I am trying to understand the difference between answer A (the correct one) and answer E on question #19. I can see why answer A would qualify as a must be true answer. However, answer E would seem to be a must be true just based on the fact that the scenario on floor 3 only allows for there to be a sports car in that slot since a family car requires a research model on that floor, which there isn't.

Or is it that the logic does not allow for assumptions to be made? As in there could be another kind of car on that floor that is not a family or a sports car and we are just to go by the conditional rules that were established in the local rules in the stimulus and any initial global rules that were not changed ?

Thank you,

Lauren

Get the most out of your LSAT Prep Plus subscription.

Analyze and track your performance with our Testing and Analytics Package.